Half Range Sine SeriesComputation Of full range fourier seriesFourier Series of The Sine FunctionFourier series with half rangeFourier sine series of $f = cos x$Find the half-range Fourier series expansion of $f(x) = cos(x)$Fourier sine series for $x^3$Half range Fourier sine series of cos(x) on 0 < x < $fracpi2$Find the Fourier series of rectified sine waveFourier Series Expansion for Half-Wave Sine ProblemFourier series of $left|xright|$

One verb to replace 'be a member of' a club

What is an equivalently powerful replacement spell for the Yuan-Ti's Suggestion spell?

Unlock My Phone! February 2018

Should I tell management that I intend to leave due to bad software development practices?

In the UK, is it possible to get a referendum by a court decision?

files created then deleted at every second in tmp directory

What is the most common color to indicate the input-field is disabled?

How obscure is the use of 令 in 令和?

Why was Sir Cadogan fired?

How to coordinate airplane tickets?

How can I deal with my CEO asking me to hire someone with a higher salary than me, a co-founder?

Why is it a bad idea to hire a hitman to eliminate most corrupt politicians?

Could the museum Saturn V's be refitted for one more flight?

Does Dispel Magic work on Tiny Hut?

What is a Samsaran Word™?

Did 'Cinema Songs' exist during Hiranyakshipu's time?

GFCI outlets - can they be repaired? Are they really needed at the end of a circuit?

What is the opposite of "eschatology"?

Was the Stack Exchange "Happy April Fools" page fitting with the '90's code?

Can a virus destroy the BIOS of a modern computer?

How can saying a song's name be a copyright violation?

What are the G forces leaving Earth orbit?

Why were 5.25" floppy drives cheaper than 8"?

What Exploit Are These User Agents Trying to Use?



Half Range Sine Series


Computation Of full range fourier seriesFourier Series of The Sine FunctionFourier series with half rangeFourier sine series of $f = cos x$Find the half-range Fourier series expansion of $f(x) = cos(x)$Fourier sine series for $x^3$Half range Fourier sine series of cos(x) on 0 < x < $fracpi2$Find the Fourier series of rectified sine waveFourier Series Expansion for Half-Wave Sine ProblemFourier series of $left|xright|$













4












$begingroup$


Question:
It is known that $f(x)=(x−4)^2$ for all $xin [0,4]$.



Compute the half range sine series expansion for $f(x)$.



My answer :



Half range series: $p=8$, $l=4$, $a_0=a_n=0$.



$$b_n=frac2Lint_0^Lf(x)sinleft(fracnpi xLright)d(x)=frac24int_0^4(x-4)^2sinleft(fracnpi x4right)d(x)$$



Partial Differentiation
Let $u=(x-4)^2$; $du=2(x-4)dx$; $v=frac-4npicos(fracnpi x4)$



beginalign
b_n&=frac12[frac-4npicos(fracnpi x4)(x-4)^2+frac8npiint(x-4)cos(fracnpi x4)d(x)]|^4_0\
&=frac12[frac-4npicos(fracnpi x4)(x-4)^2+frac8npi[frac4npisin(fracnpi x4)(x-4)-frac4npiintsin(fracnpi x4)d(x)]]|^4_0\
&=frac12[frac-4npicos(fracnpi x4)(x-4)^2$+$frac8npi[frac4npisin(fracnpi x4)(x-4)-frac4npi(frac-4npicosfracnpi x4)]|^4_0\
&=frac12[frac-4npicos(fracnpi x4)(x-4)^2$+$frac8npi(frac16n^2pi^2cosfracnpi x4)]|^4_0
endalign



we know $cosnpi=(-1)^n$



$frac12[(0+frac128(-1)^nn^3pi^3)-(-frac64npi+frac128n^3pi^3)$



$b_n=frac64(-1)^nn^3pi^3-frac64n^3pi^3+frac32npi$



My teacher told me my $b_n$ is wrong and it seems that my working also looks like wrong. Can someone please help me to recalculate my $b_n$ please. Just help me once only guys.If I get this right I would able to get full marks.I would be very very grateful to you guys if you guys able to help me. If can suggest me any ideas or simpler way to solve it if you can.Thanks










share|cite|improve this question











$endgroup$











  • $begingroup$
    Should $b_n$ be the integral from $-L$ to $L$ or the integral from $0$ to $L$ because it switches from the first line to the second line without any multiplicative factor.
    $endgroup$
    – Euler....IS_ALIVE
    Sep 9 '12 at 6:40










  • $begingroup$
    O to L. It alwys like that for Half Range Sine Series
    $endgroup$
    – David
    Sep 9 '12 at 6:48










  • $begingroup$
    But you wrote it as $-L$ to $L$ in your first definition of $b_n$....
    $endgroup$
    – Euler....IS_ALIVE
    Sep 9 '12 at 6:49










  • $begingroup$
    this is the link for reference intmath.com/fourier-series/4-fourier-half-range-functions.php
    $endgroup$
    – David
    Sep 9 '12 at 6:49










  • $begingroup$
    I edited already
    $endgroup$
    – David
    Sep 9 '12 at 6:50















4












$begingroup$


Question:
It is known that $f(x)=(x−4)^2$ for all $xin [0,4]$.



Compute the half range sine series expansion for $f(x)$.



My answer :



Half range series: $p=8$, $l=4$, $a_0=a_n=0$.



$$b_n=frac2Lint_0^Lf(x)sinleft(fracnpi xLright)d(x)=frac24int_0^4(x-4)^2sinleft(fracnpi x4right)d(x)$$



Partial Differentiation
Let $u=(x-4)^2$; $du=2(x-4)dx$; $v=frac-4npicos(fracnpi x4)$



beginalign
b_n&=frac12[frac-4npicos(fracnpi x4)(x-4)^2+frac8npiint(x-4)cos(fracnpi x4)d(x)]|^4_0\
&=frac12[frac-4npicos(fracnpi x4)(x-4)^2+frac8npi[frac4npisin(fracnpi x4)(x-4)-frac4npiintsin(fracnpi x4)d(x)]]|^4_0\
&=frac12[frac-4npicos(fracnpi x4)(x-4)^2$+$frac8npi[frac4npisin(fracnpi x4)(x-4)-frac4npi(frac-4npicosfracnpi x4)]|^4_0\
&=frac12[frac-4npicos(fracnpi x4)(x-4)^2$+$frac8npi(frac16n^2pi^2cosfracnpi x4)]|^4_0
endalign



we know $cosnpi=(-1)^n$



$frac12[(0+frac128(-1)^nn^3pi^3)-(-frac64npi+frac128n^3pi^3)$



$b_n=frac64(-1)^nn^3pi^3-frac64n^3pi^3+frac32npi$



My teacher told me my $b_n$ is wrong and it seems that my working also looks like wrong. Can someone please help me to recalculate my $b_n$ please. Just help me once only guys.If I get this right I would able to get full marks.I would be very very grateful to you guys if you guys able to help me. If can suggest me any ideas or simpler way to solve it if you can.Thanks










share|cite|improve this question











$endgroup$











  • $begingroup$
    Should $b_n$ be the integral from $-L$ to $L$ or the integral from $0$ to $L$ because it switches from the first line to the second line without any multiplicative factor.
    $endgroup$
    – Euler....IS_ALIVE
    Sep 9 '12 at 6:40










  • $begingroup$
    O to L. It alwys like that for Half Range Sine Series
    $endgroup$
    – David
    Sep 9 '12 at 6:48










  • $begingroup$
    But you wrote it as $-L$ to $L$ in your first definition of $b_n$....
    $endgroup$
    – Euler....IS_ALIVE
    Sep 9 '12 at 6:49










  • $begingroup$
    this is the link for reference intmath.com/fourier-series/4-fourier-half-range-functions.php
    $endgroup$
    – David
    Sep 9 '12 at 6:49










  • $begingroup$
    I edited already
    $endgroup$
    – David
    Sep 9 '12 at 6:50













4












4








4





$begingroup$


Question:
It is known that $f(x)=(x−4)^2$ for all $xin [0,4]$.



Compute the half range sine series expansion for $f(x)$.



My answer :



Half range series: $p=8$, $l=4$, $a_0=a_n=0$.



$$b_n=frac2Lint_0^Lf(x)sinleft(fracnpi xLright)d(x)=frac24int_0^4(x-4)^2sinleft(fracnpi x4right)d(x)$$



Partial Differentiation
Let $u=(x-4)^2$; $du=2(x-4)dx$; $v=frac-4npicos(fracnpi x4)$



beginalign
b_n&=frac12[frac-4npicos(fracnpi x4)(x-4)^2+frac8npiint(x-4)cos(fracnpi x4)d(x)]|^4_0\
&=frac12[frac-4npicos(fracnpi x4)(x-4)^2+frac8npi[frac4npisin(fracnpi x4)(x-4)-frac4npiintsin(fracnpi x4)d(x)]]|^4_0\
&=frac12[frac-4npicos(fracnpi x4)(x-4)^2$+$frac8npi[frac4npisin(fracnpi x4)(x-4)-frac4npi(frac-4npicosfracnpi x4)]|^4_0\
&=frac12[frac-4npicos(fracnpi x4)(x-4)^2$+$frac8npi(frac16n^2pi^2cosfracnpi x4)]|^4_0
endalign



we know $cosnpi=(-1)^n$



$frac12[(0+frac128(-1)^nn^3pi^3)-(-frac64npi+frac128n^3pi^3)$



$b_n=frac64(-1)^nn^3pi^3-frac64n^3pi^3+frac32npi$



My teacher told me my $b_n$ is wrong and it seems that my working also looks like wrong. Can someone please help me to recalculate my $b_n$ please. Just help me once only guys.If I get this right I would able to get full marks.I would be very very grateful to you guys if you guys able to help me. If can suggest me any ideas or simpler way to solve it if you can.Thanks










share|cite|improve this question











$endgroup$




Question:
It is known that $f(x)=(x−4)^2$ for all $xin [0,4]$.



Compute the half range sine series expansion for $f(x)$.



My answer :



Half range series: $p=8$, $l=4$, $a_0=a_n=0$.



$$b_n=frac2Lint_0^Lf(x)sinleft(fracnpi xLright)d(x)=frac24int_0^4(x-4)^2sinleft(fracnpi x4right)d(x)$$



Partial Differentiation
Let $u=(x-4)^2$; $du=2(x-4)dx$; $v=frac-4npicos(fracnpi x4)$



beginalign
b_n&=frac12[frac-4npicos(fracnpi x4)(x-4)^2+frac8npiint(x-4)cos(fracnpi x4)d(x)]|^4_0\
&=frac12[frac-4npicos(fracnpi x4)(x-4)^2+frac8npi[frac4npisin(fracnpi x4)(x-4)-frac4npiintsin(fracnpi x4)d(x)]]|^4_0\
&=frac12[frac-4npicos(fracnpi x4)(x-4)^2$+$frac8npi[frac4npisin(fracnpi x4)(x-4)-frac4npi(frac-4npicosfracnpi x4)]|^4_0\
&=frac12[frac-4npicos(fracnpi x4)(x-4)^2$+$frac8npi(frac16n^2pi^2cosfracnpi x4)]|^4_0
endalign



we know $cosnpi=(-1)^n$



$frac12[(0+frac128(-1)^nn^3pi^3)-(-frac64npi+frac128n^3pi^3)$



$b_n=frac64(-1)^nn^3pi^3-frac64n^3pi^3+frac32npi$



My teacher told me my $b_n$ is wrong and it seems that my working also looks like wrong. Can someone please help me to recalculate my $b_n$ please. Just help me once only guys.If I get this right I would able to get full marks.I would be very very grateful to you guys if you guys able to help me. If can suggest me any ideas or simpler way to solve it if you can.Thanks







calculus real-analysis fourier-series






share|cite|improve this question















share|cite|improve this question













share|cite|improve this question




share|cite|improve this question








edited Sep 9 '12 at 8:20









Davide Giraudo

128k17156268




128k17156268










asked Sep 9 '12 at 6:31









DavidDavid

8019




8019











  • $begingroup$
    Should $b_n$ be the integral from $-L$ to $L$ or the integral from $0$ to $L$ because it switches from the first line to the second line without any multiplicative factor.
    $endgroup$
    – Euler....IS_ALIVE
    Sep 9 '12 at 6:40










  • $begingroup$
    O to L. It alwys like that for Half Range Sine Series
    $endgroup$
    – David
    Sep 9 '12 at 6:48










  • $begingroup$
    But you wrote it as $-L$ to $L$ in your first definition of $b_n$....
    $endgroup$
    – Euler....IS_ALIVE
    Sep 9 '12 at 6:49










  • $begingroup$
    this is the link for reference intmath.com/fourier-series/4-fourier-half-range-functions.php
    $endgroup$
    – David
    Sep 9 '12 at 6:49










  • $begingroup$
    I edited already
    $endgroup$
    – David
    Sep 9 '12 at 6:50
















  • $begingroup$
    Should $b_n$ be the integral from $-L$ to $L$ or the integral from $0$ to $L$ because it switches from the first line to the second line without any multiplicative factor.
    $endgroup$
    – Euler....IS_ALIVE
    Sep 9 '12 at 6:40










  • $begingroup$
    O to L. It alwys like that for Half Range Sine Series
    $endgroup$
    – David
    Sep 9 '12 at 6:48










  • $begingroup$
    But you wrote it as $-L$ to $L$ in your first definition of $b_n$....
    $endgroup$
    – Euler....IS_ALIVE
    Sep 9 '12 at 6:49










  • $begingroup$
    this is the link for reference intmath.com/fourier-series/4-fourier-half-range-functions.php
    $endgroup$
    – David
    Sep 9 '12 at 6:49










  • $begingroup$
    I edited already
    $endgroup$
    – David
    Sep 9 '12 at 6:50















$begingroup$
Should $b_n$ be the integral from $-L$ to $L$ or the integral from $0$ to $L$ because it switches from the first line to the second line without any multiplicative factor.
$endgroup$
– Euler....IS_ALIVE
Sep 9 '12 at 6:40




$begingroup$
Should $b_n$ be the integral from $-L$ to $L$ or the integral from $0$ to $L$ because it switches from the first line to the second line without any multiplicative factor.
$endgroup$
– Euler....IS_ALIVE
Sep 9 '12 at 6:40












$begingroup$
O to L. It alwys like that for Half Range Sine Series
$endgroup$
– David
Sep 9 '12 at 6:48




$begingroup$
O to L. It alwys like that for Half Range Sine Series
$endgroup$
– David
Sep 9 '12 at 6:48












$begingroup$
But you wrote it as $-L$ to $L$ in your first definition of $b_n$....
$endgroup$
– Euler....IS_ALIVE
Sep 9 '12 at 6:49




$begingroup$
But you wrote it as $-L$ to $L$ in your first definition of $b_n$....
$endgroup$
– Euler....IS_ALIVE
Sep 9 '12 at 6:49












$begingroup$
this is the link for reference intmath.com/fourier-series/4-fourier-half-range-functions.php
$endgroup$
– David
Sep 9 '12 at 6:49




$begingroup$
this is the link for reference intmath.com/fourier-series/4-fourier-half-range-functions.php
$endgroup$
– David
Sep 9 '12 at 6:49












$begingroup$
I edited already
$endgroup$
– David
Sep 9 '12 at 6:50




$begingroup$
I edited already
$endgroup$
– David
Sep 9 '12 at 6:50










1 Answer
1






active

oldest

votes


















1












$begingroup$

The integral defining your $b_n$ is correct, but the final answer is wrong. It differs from mine by $64over npi$.



Here's my work from Mathematica:



Mathematica graphics



Here's graphical verification that we are indeed computing the series correctly (I took the 5th, 10th, and 50th partial sum, respectively):



Mathematica graphics






share|cite|improve this answer









$endgroup$












  • $begingroup$
    Hi JohnD. I check out my lecturer he said mine is correct now. Maybe yours is correct too but in different form I think
    $endgroup$
    – David
    May 29 '13 at 5:45










  • $begingroup$
    Yes, they are algebraically equivalent.
    $endgroup$
    – JohnD
    May 29 '13 at 15:06











Your Answer





StackExchange.ifUsing("editor", function ()
return StackExchange.using("mathjaxEditing", function ()
StackExchange.MarkdownEditor.creationCallbacks.add(function (editor, postfix)
StackExchange.mathjaxEditing.prepareWmdForMathJax(editor, postfix, [["$", "$"], ["\\(","\\)"]]);
);
);
, "mathjax-editing");

StackExchange.ready(function()
var channelOptions =
tags: "".split(" "),
id: "69"
;
initTagRenderer("".split(" "), "".split(" "), channelOptions);

StackExchange.using("externalEditor", function()
// Have to fire editor after snippets, if snippets enabled
if (StackExchange.settings.snippets.snippetsEnabled)
StackExchange.using("snippets", function()
createEditor();
);

else
createEditor();

);

function createEditor()
StackExchange.prepareEditor(
heartbeatType: 'answer',
autoActivateHeartbeat: false,
convertImagesToLinks: true,
noModals: true,
showLowRepImageUploadWarning: true,
reputationToPostImages: 10,
bindNavPrevention: true,
postfix: "",
imageUploader:
brandingHtml: "Powered by u003ca class="icon-imgur-white" href="https://imgur.com/"u003eu003c/au003e",
contentPolicyHtml: "User contributions licensed under u003ca href="https://creativecommons.org/licenses/by-sa/3.0/"u003ecc by-sa 3.0 with attribution requiredu003c/au003e u003ca href="https://stackoverflow.com/legal/content-policy"u003e(content policy)u003c/au003e",
allowUrls: true
,
noCode: true, onDemand: true,
discardSelector: ".discard-answer"
,immediatelyShowMarkdownHelp:true
);



);













draft saved

draft discarded


















StackExchange.ready(
function ()
StackExchange.openid.initPostLogin('.new-post-login', 'https%3a%2f%2fmath.stackexchange.com%2fquestions%2f193055%2fhalf-range-sine-series%23new-answer', 'question_page');

);

Post as a guest















Required, but never shown

























1 Answer
1






active

oldest

votes








1 Answer
1






active

oldest

votes









active

oldest

votes






active

oldest

votes









1












$begingroup$

The integral defining your $b_n$ is correct, but the final answer is wrong. It differs from mine by $64over npi$.



Here's my work from Mathematica:



Mathematica graphics



Here's graphical verification that we are indeed computing the series correctly (I took the 5th, 10th, and 50th partial sum, respectively):



Mathematica graphics






share|cite|improve this answer









$endgroup$












  • $begingroup$
    Hi JohnD. I check out my lecturer he said mine is correct now. Maybe yours is correct too but in different form I think
    $endgroup$
    – David
    May 29 '13 at 5:45










  • $begingroup$
    Yes, they are algebraically equivalent.
    $endgroup$
    – JohnD
    May 29 '13 at 15:06















1












$begingroup$

The integral defining your $b_n$ is correct, but the final answer is wrong. It differs from mine by $64over npi$.



Here's my work from Mathematica:



Mathematica graphics



Here's graphical verification that we are indeed computing the series correctly (I took the 5th, 10th, and 50th partial sum, respectively):



Mathematica graphics






share|cite|improve this answer









$endgroup$












  • $begingroup$
    Hi JohnD. I check out my lecturer he said mine is correct now. Maybe yours is correct too but in different form I think
    $endgroup$
    – David
    May 29 '13 at 5:45










  • $begingroup$
    Yes, they are algebraically equivalent.
    $endgroup$
    – JohnD
    May 29 '13 at 15:06













1












1








1





$begingroup$

The integral defining your $b_n$ is correct, but the final answer is wrong. It differs from mine by $64over npi$.



Here's my work from Mathematica:



Mathematica graphics



Here's graphical verification that we are indeed computing the series correctly (I took the 5th, 10th, and 50th partial sum, respectively):



Mathematica graphics






share|cite|improve this answer









$endgroup$



The integral defining your $b_n$ is correct, but the final answer is wrong. It differs from mine by $64over npi$.



Here's my work from Mathematica:



Mathematica graphics



Here's graphical verification that we are indeed computing the series correctly (I took the 5th, 10th, and 50th partial sum, respectively):



Mathematica graphics







share|cite|improve this answer












share|cite|improve this answer



share|cite|improve this answer










answered Dec 20 '12 at 7:00









JohnDJohnD

12k32158




12k32158











  • $begingroup$
    Hi JohnD. I check out my lecturer he said mine is correct now. Maybe yours is correct too but in different form I think
    $endgroup$
    – David
    May 29 '13 at 5:45










  • $begingroup$
    Yes, they are algebraically equivalent.
    $endgroup$
    – JohnD
    May 29 '13 at 15:06
















  • $begingroup$
    Hi JohnD. I check out my lecturer he said mine is correct now. Maybe yours is correct too but in different form I think
    $endgroup$
    – David
    May 29 '13 at 5:45










  • $begingroup$
    Yes, they are algebraically equivalent.
    $endgroup$
    – JohnD
    May 29 '13 at 15:06















$begingroup$
Hi JohnD. I check out my lecturer he said mine is correct now. Maybe yours is correct too but in different form I think
$endgroup$
– David
May 29 '13 at 5:45




$begingroup$
Hi JohnD. I check out my lecturer he said mine is correct now. Maybe yours is correct too but in different form I think
$endgroup$
– David
May 29 '13 at 5:45












$begingroup$
Yes, they are algebraically equivalent.
$endgroup$
– JohnD
May 29 '13 at 15:06




$begingroup$
Yes, they are algebraically equivalent.
$endgroup$
– JohnD
May 29 '13 at 15:06

















draft saved

draft discarded
















































Thanks for contributing an answer to Mathematics Stack Exchange!


  • Please be sure to answer the question. Provide details and share your research!

But avoid


  • Asking for help, clarification, or responding to other answers.

  • Making statements based on opinion; back them up with references or personal experience.

Use MathJax to format equations. MathJax reference.


To learn more, see our tips on writing great answers.




draft saved


draft discarded














StackExchange.ready(
function ()
StackExchange.openid.initPostLogin('.new-post-login', 'https%3a%2f%2fmath.stackexchange.com%2fquestions%2f193055%2fhalf-range-sine-series%23new-answer', 'question_page');

);

Post as a guest















Required, but never shown





















































Required, but never shown














Required, but never shown












Required, but never shown







Required, but never shown

































Required, but never shown














Required, but never shown












Required, but never shown







Required, but never shown







Popular posts from this blog

Triangular numbers and gcdProving sum of a set is $0 pmod n$ if $n$ is odd, or $fracn2 pmod n$ if $n$ is even?Is greatest common divisor of two numbers really their smallest linear combination?GCD, LCM RelationshipProve a set of nonnegative integers with greatest common divisor 1 and closed under addition has all but finite many nonnegative integers.all pairs of a and b in an equation containing gcdTriangular Numbers Modulo $k$ - Hit All Values?Understanding the Existence and Uniqueness of the GCDGCD and LCM with logical symbolsThe greatest common divisor of two positive integers less than 100 is equal to 3. Their least common multiple is twelve times one of the integers.Suppose that for all integers $x$, $x|a$ and $x|b$ if and only if $x|c$. Then $c = gcd(a,b)$Which is the gcd of 2 numbers which are multiplied and the result is 600000?

Ingelân Ynhâld Etymology | Geografy | Skiednis | Polityk en bestjoer | Ekonomy | Demografy | Kultuer | Klimaat | Sjoch ek | Keppelings om utens | Boarnen, noaten en referinsjes Navigaasjemenuwww.gov.ukOffisjele webside fan it regear fan it Feriene KeninkrykOffisjele webside fan it Britske FerkearsburoNederlânsktalige ynformaasje fan it Britske FerkearsburoOffisjele webside fan English Heritage, de organisaasje dy't him ynset foar it behâld fan it Ingelske kultuergoedYnwennertallen fan alle Britske stêden út 'e folkstelling fan 2011Notes en References, op dizze sideEngland

Հադիս Բովանդակություն Անվանում և նշանակություն | Դասակարգում | Աղբյուրներ | Նավարկման ցանկ